Quantcast
  • Register
PhysicsOverflow is a next-generation academic platform for physicists and astronomers, including a community peer review system and a postgraduate-level discussion forum analogous to MathOverflow.

Welcome to PhysicsOverflow! PhysicsOverflow is an open platform for community peer review and graduate-level Physics discussion.

Please help promote PhysicsOverflow ads elsewhere if you like it.

News

PO is now at the Physics Department of Bielefeld University!

New printer friendly PO pages!

Migration to Bielefeld University was successful!

Please vote for this year's PhysicsOverflow ads!

Please do help out in categorising submissions. Submit a paper to PhysicsOverflow!

... see more

Tools for paper authors

Submit paper
Claim Paper Authorship

Tools for SE users

Search User
Reclaim SE Account
Request Account Merger
Nativise imported posts
Claim post (deleted users)
Import SE post

Users whose questions have been imported from Physics Stack Exchange, Theoretical Physics Stack Exchange, or any other Stack Exchange site are kindly requested to reclaim their account and not to register as a new user.

Public \(\beta\) tools

Report a bug with a feature
Request a new functionality
404 page design
Send feedback

Attributions

(propose a free ad)

Site Statistics

205 submissions , 163 unreviewed
5,047 questions , 2,200 unanswered
5,345 answers , 22,709 comments
1,470 users with positive rep
816 active unimported users
More ...

  How to derive the Goldstone-Wilczek current?

+ 2 like - 0 dislike
1396 views

I am reading this [Goldstone-Wilczek's celebrated] paper [1] on fractional quantum number. In particular they derived for the following Dirac Lagrangian ($\phi_1$ and $\phi_2$ are scalar fields)

$\mathscr{L}=i\bar{\psi}\gamma^\mu\partial_\mu\psi+g\bar{\psi}(\phi_1+i\gamma_5\phi_2)\psi$

that the current expectation reads ($\theta=\tan^{-1}(\phi_2/\phi_1)$)

$\langle j^\mu\rangle=\frac{1}{2\pi}\epsilon^{\mu\nu}\partial_\nu\theta$,

which is equation (2). It is possible to understand it via dimensional reduction or bosonization. But I would like to understand it in a field-theoretic way. In particular I have two questions:

1. How to get the Feynman diagram in Fig.3? (I am aware of the similar question in [here][2] but it does not answer my question, and the Feynman diagram below is from that thread) (For clarity, the curly line is a current, the solid line is a fermion and the dash-line is  a scalar)
2. How to calculate this Feynman diagram?

I have looked on the internet and at quite a few references, and could not figure it out. Any help is appreciated.

  [1]: http://journals.aps.org/prl/abstract/10.1103/PhysRevLett.47.986
  [2]: http://physics.stackexchange.com/questions/116931/tadpole-diagram-and-vacuum

asked Dec 18, 2016 in Theoretical Physics by fagd (20 points) [ revision history ]
recategorized Dec 18, 2016 by Dilaton

1 Answer

+ 3 like - 0 dislike


It is interesting to view your question above not only in light of the Goldstone-Wilczek (G-W) approach (G-W has provided a method for computing the fermion charge induced by a classical profile), but also by computing  $1/2$-fermion charge found by [Jackiw-Rebbi](http://journals.aps.org/prd/abstract/10.1103/PhysRevD.13.3398) using G-W method. For simplicity, let us consider the 1+1D case, and let us consider the $Z_2$ domain wall and the $1/2$-charge found by Jackiw-Rebbi. The construction, valid for 1+1D systems, works as follows.


Consider a Lagrangian describing spinless fermions $\psi(x)$ coupled to a classical background profile $\lambda(x)$
via a term $\lambda\,\psi^{\dagger}\sigma_{3} \psi$. In the high temperature phase, the v.e.v. of $\lambda$ is zero and no mass is generated
for the fermions. In the low temperature phase, the $\lambda$ acquires two degenerate vacuum values $\pm  \langle \lambda \rangle$ 
that are related by a ${Z}_2$ symmetry. Generically we have
$$
\langle \lambda \rangle\,\cos\big( \phi(x) - \theta(x) \big),
$$
where we use the bosonization dictionary 
$
\psi^{\dagger}\sigma_{3} \psi
\rightarrow
\cos(\phi(x))
$
and a phase change $\Delta\theta = \pi$ captures the existence of a domain wall
separating regions with opposite values of the v.e.v. of $\lambda$.
From the fact that the fermion density 
$$
\rho(x)
=
\psi^{\dagger}(x)\psi(x) 

\frac{1}{2\pi} \partial_{x} \phi(x)
,$$ 
and the current 
$$J^\mu=\psi^{\dagger}\gamma^\mu\psi =\frac{1}{2\pi}\epsilon^{\mu \nu} \partial_\nu \phi,$$
it follows that the induced charge $Q_{\text{dw}}$ on the kink by a domain wall is
$$
Q_{\text{dw}} 

\int^{x_0 + \varepsilon}_{x_0 - \varepsilon}\,dx\,\rho(x)
=
\int^{x_0 + \varepsilon}_{x_0 - \varepsilon}\,dx\,\frac{1}{2\pi} \partial_{x} \phi(x)
=
\frac{1}{2\pi} \pi = 
\frac{1}{2},
$$
where $x_0$ denotes the center of the domain wall.

You can try to extend to other dimensions, but then you may need to be careful and you may not be able to use the bosonization. 

See more details of the derivation here in the [page 13](https://arxiv.org/abs/1403.5256) of [this paper](http://journals.aps.org/prb/abstract/10.1103/PhysRevB.91.195134).

answered Dec 20, 2016 by wonderich (1,500 points) [ revision history ]

Hi Idear,

Thanks for the reply. Indeed G-W's current becomes very clear in the language of bosonization. It is possible as well to come to the same formula via dimension reduction, see for example http://journals.aps.org/prb/abstract/10.1103/PhysRevB.78.195424. Basically the angle $\theta$ in G-W's current correspond to the gauge field in the Quantum Hall response, which is kind of an "extension" to higher dimensions. 

But I am just curious how G-W come up with this diagram in the first place? Why we have to insert five scalar fields in the diagram, not one or two for example? 

It seems related to the soliton background, which I could not find a suitable reference... Any idea?

1) Yes, the dimensional reduction may work.

2) They Feynman diagram shall come up in any number of scaler field background insertion. The 5 dashed lines are just schematic. You can consider just 1 dashed line is good enough. I remember that I compute it before directly through this Feynman diagram, but it is a bit long calculation. I have to dig it out, it is somewhere in my files. In any case of two approaches, the result shall be the same.

That would make much more sense, I was struggling to understand why five insertion gives only one derivative of the scalar field... Having say that, it will be great if you could sketch your calculation here, just briefly. Thank you.

Your answer

Please use answers only to (at least partly) answer questions. To comment, discuss, or ask for clarification, leave a comment instead.
To mask links under text, please type your text, highlight it, and click the "link" button. You can then enter your link URL.
Please consult the FAQ for as to how to format your post.
This is the answer box; if you want to write a comment instead, please use the 'add comment' button.
Live preview (may slow down editor)   Preview
Your name to display (optional):
Privacy: Your email address will only be used for sending these notifications.
Anti-spam verification:
If you are a human please identify the position of the character covered by the symbol $\varnothing$ in the following word:
p$\hbar$ysicsOve$\varnothing$flow
Then drag the red bullet below over the corresponding character of our banner. When you drop it there, the bullet changes to green (on slow internet connections after a few seconds).
Please complete the anti-spam verification




user contributions licensed under cc by-sa 3.0 with attribution required

Your rights
...